Frank will choose 7 colors to use for an art project. If there are 10 colors to choose from, how many different color combinations are possible?

Answers

Answer 1

120

Explanation:

total colours = 10

number of colours to be chosen = 7

We apply combination

The different color combinations possible:

[tex]^{10}C_7=\frac{10!}{(10-7)!7!}[/tex][tex]\begin{gathered} =\frac{10!}{3!7!}=\frac{10\times9\times8\times7!}{3\times2\times1\times7!} \\ =\frac{720}{6} \\ \text{= 120} \end{gathered}[/tex]

The different color combinations possible is 120


Related Questions

given two circles (all circles are similar) , with circumferences of 30cm and 12cm each, find the ratio of their areas. state answer as fraction.

Answers

The circumference of a circle is given by the following formula

[tex]C=2\pi r[/tex]

where r represents the radius.

The ratio between two circumferences is equal to the ratio of the radius.

[tex]\frac{C_1}{C_2}=\frac{2\pi r_1}{2\pi r_2}=\frac{r_1}{r_2}[/tex]

The area of a circle is given by the following formula

[tex]A=\pi r^2[/tex]

Then, the ratio between two circle areas is equal to the square of the ratio of the radius, which is the square of the ratio between the circumferences.

[tex]\frac{A_1}{A_2}=\frac{\pi r_1^2}{\pi r_2^2}=(\frac{r_1}{r_2})^2=(\frac{C_1}{C_2})^2[/tex]

Then, applying this relation in our problem, the ratio between the areas is:

[tex]\frac{A_1}{A_2}=(\frac{30}{12})^2=\frac{25}{4}[/tex]

The ratio between the areas is 25/4.

Find the probability of getting 4 aces when 5 cards are drawn from an ordinary deck of cards

Answers

First, let's calculate the number of different hands of 5 cards that can be made, using a combination of 52 choose 5:

(a standard deck card has 52 cards)

[tex]C\left(52,5\right)=\frac{52!}{5!\left(52-5\right)!}=\frac{52\cdot51\operatorname{\cdot}50\operatorname{\cdot}49\operatorname{\cdot}48\operatorname{\cdot}47!}{5\operatorname{\cdot}4\operatorname{\cdot}3\operatorname{\cdot}2\operatorname{\cdot}47!}=\frac{52\cdot51\operatorname{\cdot}50\operatorname{\cdot}49\operatorname{\cdot}48}{120}=2,598,960[/tex]

Now, let's calculate the number of hands that have 4 aces. Since the fifth card can be any of the remaining 48 cards after picking the 4 aces, there are 48 possible hands that have 4 aces.

Then, the probability of having a hand with 4 aces is given by the division of these 48 possible hands over the total number of possible hands of 5 cards:

[tex]P=\frac{48}{2598960}=\frac{1}{54145}[/tex]

The probability is 1/54145.

Find the missing parts of the triangle. Round to the nearest tenth when necessary or to the nearest minute as appropriate.C=111.1°a=7.1mb=9.6mOption 1: No triangle satisfies the given conditions.Option 2: c=19.6m, A=26.8°, B=42.1°Option 3: c=16.7m, A=30.8°, B=38.1°Option 4: c=13.8m, A=28.8°, B=40.1°

Answers

Answer: Option 4: c=13.8m, A=28.8°, B=40.1°

Explanation:

From the information given,

the known sides are a = 7.1 and b = 9.6

the known angle is C = 111.1

We would find side c by applying the cosine rule which is expressed as

c^2 = a^2 + b^2 - 2abCosC

By substituting the given values into the formula,

c^2 = 7.1^2 + 9.6^2 - 2 x 7.1 x 9.6Cos111.1

c^2 = 50.41 + 92.16 - 136.32Cos111.1

c^2 = 142.57 - 136.32Cos111.1 = 191.6448

c = √191.6448 = 13.8436

c = 13.8

To find angle A, we would apply the sine rule which is expressed as

a/SinA = c/SinC

Thus,

7.1/SinA = 13.8436/Sin 111.1

By cross multiplying, we have

13.8436SinA = 7.1Sin111.1

SinA = 7.1Sin111.1/13.8436 = 0.4785

Taking the sine inverse of 0.4785,

A = 28.8

Recall, the sum of the angles in a triangle is 180. Thus,

A + B + C = 180

28.8 + B + 111.1 = 180

139.9 + B = 180

B = 180 - 139.9

B = 40.1

Option 4: c=13.8m, A=28.8°, B=40.1°

In 2009, there were 6.1 million females enrolled in degree granting institutions of higher education. over the next several years this number increased at a rate of 400,000 per year. estimate the number of females enrolled in 2024. y = ______ millionthe equation of the line that models this information is;y = 0.4t + 6.1Determine what year 12.9 million females will be enrolled.

Answers

Notice that

400,000 = 0.4 million

That's why the equation that models that information has the factor 0.4, since it expresses the result in millions of females.

Now, we need to notice that t, in the expression 0.4t + 6.1, is the number of years passed since 2009. So, in the year 2024, we have:

t = 2024 - 2009 = 15

Therefore, the number of females enrolled in 2024 can be estimated to be:

y = (0.4 * 15 + 6.1) million

y = (6 + 6.1) million

y = 12.1 million

Now, to determine the year when 12.9 million females will be enrolled, we first need to find t corresponding to y = 12.9, and then add it to the year 2009.

y = 0.4t + 6.1​

12.9 = 0.4t + 6.1

12.9 - 6.1 = 0.4t

6.8 = 0.4t

t = 6.8/0.4

t = 68/4

t = 17

Therefore, the year when it happens will be:

2009 + 17 = 2026

how many pennies are in a dollar

Answers

Answer: 100

Step-by-step explanation:

$1 =100 pennies

Which point is on the circle centered at the origin with a radius of 5 units?Distance formula: Vx2 - xy)2 + (V2 - y2)?(2, 721)(2, 23)(2, 1)O (2,3)

Answers

To know if the point is on the circle, we mus calculate the distance between the point and the origin.

For the first option, we have:

- (2, √21)

and the origin

- (0, 0)

Then, we must replace the two points in the distance formula:

[tex]d=\sqrt[]{(x_2-x_1)^2+(y_2-y_1)^2}[/tex][tex]\begin{gathered} d=\sqrt[]{(0-2)^2+(0-\sqrt[]{21})^2} \\ d=\sqrt[]{4+21}=\sqrt[]{25}=5 \end{gathered}[/tex]

Knowing that the distancie is 5 we can affirm that the point is on the circle because the radius is 5.

Finally, the answer is

[tex](2,\text{ }\sqrt[]{21})[/tex]

A publisher for promising new novel figures fixed costs at $61,000 and variable cost at $1.50 for each book produced if the book is sold to distributors for $15 each how many must be produced and sold for publisher to break even?

Answers

SOLUTION

Given the question in the image, the following are the solution steps to answer the question.

STEP 1: Write the given information

[tex]\begin{gathered} For\text{ the cost price function:} \\ Fixed\text{ cost=\$61,000 = constant} \\ Variable\text{ cost = \$1.50 }\times\text{ number of books} \\ Let\text{ x be the number of books produced} \end{gathered}[/tex]

The function for the cost price becomes:

[tex]61000+1.5x[/tex]

STEP 2: Get the function for the selling price

The function for the selling price becomes:

[tex]\text{ \$}15x[/tex]

STEP 3: Calculate the number of books required to break even

To get the breakeven, the cost price will be equal to selling price. Therefore,

[tex]\begin{gathered} 61000+1.5x=15x \\ Subtract\text{ 1.5x from both sides} \\ 61000+1.5x-1.5x=15x-1.5x \\ 61000=13.5x \\ Divide\text{ both sides by 13.5} \\ \frac{61000}{13.5}=\frac{13.5x}{13.5} \\ 4518.518519=x \\ x\approx4519 \end{gathered}[/tex]

Hence, the number of books that must be produced and sold to get a breakeven is approximately 4519

Use the formula for n^P_r to evaluate the following expression.

Answers

Use the following formula:

[tex]_nP_r=\frac{n!}{(n-r)!}[/tex]

Then, for 11P6:

[tex]\begin{gathered} _{11}P_6=\frac{11!}{(11-6)!}=\frac{11!}{5!}=\frac{5!\cdot6\cdot7\cdot8\cdot9\cdot10\cdot11}{5!} \\ _{11}P_6=6\cdot7\cdot8\cdot9\cdot10\cdot11=332640 \end{gathered}[/tex]

Hence, the result is 332640

Answer two questions about Equations A and B
A. 2r-1= 5x
B. -1 = 3x
1) How can we get Equation B from Equation A?

Choose 1 answer:

Add/subtract the same quantity to/from both sides

Add/subtract a quantity to/from only one side

Rewrite one side (or both) by combining like terms

Rewrite one side (or both) using the distributive property

Answers

In the given equation A, we can (A) subtract the same quantity from both sides.

What are equations?In a mathematical equation, the equals sign is used to express that two expressions are equal. An equation is a mathematical statement that contains the symbol "equal to" between two expressions with identical values. Like 3x + 5 = 15, for example. There are many different types of equations, including linear, quadratic, cubic, and others. The three primary forms of linear equations are point-slope, standard, and slope-intercept.

So, obtain equation B from equation A:

Equation A: 2x - 1 = 5xEquation B: -1 = 3x

We can subtract (2x) from both sides to get equation B as follows:

2x - 1 = 5x2x - 2x - 1 = 5x - 2x-1 = 3x

Therefore, in the given equation A, we can (A) subtract the same quantity from both sides.

Know more about equations here:

https://brainly.com/question/28937794

#SPJ13

Find y if the line through (1, y) and (8, 2) has a slope of 3.

Answers

[tex](\stackrel{x_1}{1}~,~\stackrel{y_1}{y})\qquad (\stackrel{x_2}{8}~,~\stackrel{y_2}{2}) ~\hfill \stackrel{slope}{m}\implies \cfrac{\stackrel{rise} {\stackrel{y_2}{2}-\stackrel{y1}{y}}}{\underset{run} {\underset{x_2}{8}-\underset{x_1}{1}}} ~~ = ~~\stackrel{\stackrel{m}{\downarrow }}{3}\implies \cfrac{2-y}{7}=3 \\\\\\ 2-y=21\implies -y=19\implies y=\cfrac{19}{-1}\implies y=-19[/tex]

ranslateSave & Exit CertifyLesson: 10.2 Parabolas11/15Question 9 of 9, Step 1 of 1CorrectFind the equationof the parabola with the following properties. Express your answer in standard form.

Answers

Given

[tex]undefined[/tex]

Solution

Standard from of a parabola

[tex](x-H-h)^2=4p(y-k)[/tex]

Use the strategy to simplify 4/576Write the prime factorization of the radicand.442834O42/2832O 4./283²O4. 2882

Answers

To simplify the fraction we will need to facto

During Thanksgiving Break, 68% of a school's students ate green bean casserole. Out of 650 students, how many ate green bean casserole?

Answers

650 --- total

650*.68=442

442 students ate green bean casserole

.68 represents the percentage

so for example, if they asked me for 50% of 1000

we need to multiply 1000*0.5

if they asked for 60% we will multiply 1000*0.6

what is 3 8/9 + 8 1/2

Answers

[tex]\begin{gathered} 3\frac{8}{9}+8\frac{1}{2}=\frac{3\cdot9+8}{9}+\frac{2\cdot8+1}{2} \\ =\frac{27+8}{9}+\frac{16+1}{2} \\ =\frac{35}{9}+\frac{17}{2} \\ \\ =\frac{35\cdot2+17\cdot9}{9\cdot2} \\ =\frac{70+153}{18} \\ =\frac{223}{18} \\ =12\frac{7}{8} \end{gathered}[/tex]

A wheel is rotating 600 times per minute. Through how many degrees does a point in the edge of the wheel move in 1/2 seconds.

Answers

The wheel is rotating 600 times per minute, find how many times rotate in 1 second:

1 minute = 60 seconds

[tex]600\frac{times}{\min}\cdot\frac{1\min}{60s}=10\frac{times}{s}[/tex]

Then, if in 1 second it rotates 10 times in 1/2 seconds it rotates:

[tex]\frac{10\frac{times}{s}}{2}=5\text{times}[/tex]

Multiply the number of times it rotates (5 times) by 360 (a wheel has 360º)

[tex]5\text{times}\cdot\frac{360º}{1\text{time}}=1800º[/tex]Then, a point moves 1800º in 1/2 seconds

Calculate the slope of the given line using either the slope formula  m = y 2 − y 1 x 2 − x 1  or by counting  r i s e r u n . Simplify your answer. You can choose your method.

Answers

The slope of the line that passes through points (x1, y1) and (x2, y2) is computed as follows:

[tex]m=\frac{y_2-y_1}{x_2-x_1}[/tex]

Replacing with the points (-8, 3) and (0,1) we get:

[tex]m=\frac{1-3}{0-_{}(-8)}=\frac{-2}{8}=-\frac{1}{4}[/tex]

the pie chart below shows how the annual budget for general Manufacturers Incorporated is divided by department. use this chart to answer the questions

Answers

You can read a pie chart as follows

Looking at the given pie chart.

The budget for Research is arounf 1/6

The budget for Engineering is around 2/6

The budget for Support is around 1/8

The budget for media and marketing are 1/16 each

The budget for sales is around 3/16

a) The department that has one eight of the budget is Support.

b) The budgets for sales and marketing together add up to

[tex]\frac{3}{16}+\frac{1}{16}=\frac{4}{16}=\frac{1}{4}[/tex]

Multiply it by 100 to express it as a percentage

[tex]\frac{1}{4}\cdot100=25[/tex]

25% of the budget correpsonds to sales and marketing

c) The budget for media looks around one third the budget for research, to determine the percentage of budget that corresponds to media, divide the budget of research by 3

[tex]\frac{18}{3}=6[/tex]

The budget for media is 6%

Lashonda deposits $500 into an account that pays simple interest at a rate of 6% per year. How much interest will she be paid in the first 3 years?

Answers

Answer:

The amount of interest she will be paid in the first 3 years is;

[tex]\text{ \$90}[/tex]

Explanation:

Given that Lashonda deposits $500 into an account that pays simple interest at a rate of 6% per year. for the first 3 years;

[tex]\begin{gathered} \text{ Principal P = \$500} \\ \text{rate r = 6\% = 0.06} \\ \text{time t = 3 years} \end{gathered}[/tex]

Recall the simple interest formula;

[tex]i=P\times r\times t[/tex]

substituting the given values;

[tex]\begin{gathered} i=500\times0.06\times3 \\ i=\text{ \$90} \end{gathered}[/tex]

Therefore, the amount of interest she will be paid in the first 3 years is;

[tex]\text{ \$90}[/tex]

Quadrilateral ABCD is a rhombus.DA АC СBMatch the reasons that justifies the given statements.

Answers

A rhombus is a quadrilateral with 4 congruent sides.

For the Rhombus ABCD given

[tex]\begin{gathered} AB\mleft\Vert DC\text{ }\mright? \\ \\ \text{Opposite sides of a rho}mbus\text{ are parallel} \end{gathered}[/tex]

Also,

[tex]\begin{gathered} DA\cong CB \\ \text{Opposite sides of a rhombus are congruent} \end{gathered}[/tex]

Also,

[tex]\begin{gathered} <\text{ADC}\cong<\text{ABC} \\ \text{Opposite angles of a rhombus are congruent} \end{gathered}[/tex]

Find the length of the third side. If necessary, write in simplest radical form. 9 5 Submit Answer Answer:

Answers

The Pythagorean theorem states:

[tex]c^2=a^2+b^2[/tex]

where a and b are the legs and c is the hypotenuse of a right triangle.

Substituting with c = 9 and a = 5, we get:

[tex]\begin{gathered} 9^2=5^2+b^2 \\ 81=25+b^2 \\ 81-25=b^2 \\ 56=b^2 \\ \sqrt[]{56}=b \\ \sqrt[]{4\cdot14}=b \\ \sqrt[]{4}\cdot\sqrt[]{14}=b \\ 2\sqrt[]{14}=b \end{gathered}[/tex]

O A. 1376 square inchesO B. 672 square inchesO C. 1562 square inchesO D. 936 square inches

Answers

The seat back cushion is a cuboid. The surafce area can be calculated below

[tex]\begin{gathered} l=26\text{ inches} \\ h=5\text{ inches} \\ w=18\text{ inches} \\ \text{surface area=2(}lw+wh+hl\text{)} \\ \text{surface area=}2(26\times18+18\times5+5\times26) \\ \text{surface area=}2(468+90+130) \\ \text{surface area=}2\times688 \\ \text{surface area}=1376inches^2 \end{gathered}[/tex]

A linear regression model for the revenue data for a company is R=25.9t + 204 where R is total annual revenue and t is time since 1/31/02 in years.

Answers

The linear regression model is

[tex]R=25.9t+204[/tex]

Where

R is the total annual revenue (dependant variable)

t is the time, in years, since 1/31/02 (independent variable)

To predict the annual revenue for the period ending 1/31/10, the first step is to determine the value of t. Considering that t=0 is the first recorded year (1/31/02), the value of t corresponding to period 1/31/10 is the number of years passed since, including 2002, which is 9 years.

So you have to calculate R for t=9. Replace the formula with t=9 and calculate the corresponding value of R

[tex]\begin{gathered} R=25.9\cdot9+204 \\ R=437.1 \end{gathered}[/tex]

R≈437 billion dollars

I need help I already answered just to make sure

Answers

The height of the tree is 13.82 m

Step - by - Step Explanation

What to find? Height of the tree.

Given:

• Angle of elevation = 62,°

,

• Eye-level above the ground =160cm

,

• Distance away from the tree = 6.5m

We need to first sketch the problem, to have a clearer picture of the question.

Change 160cm to meter

160 cm = 160/100 = 1.6 m

Height of the tree = 1.6 + x

We need to find the value of x.

From the sketch above;

Opposite =x

Adjacent =6.5

θ= 62°

Using the trigonometric ratio;

[tex]\tan \theta=\frac{opposite\text{ }}{\text{adjacent}}[/tex]

Substitute the values.

[tex]\tan 62=\frac{x}{6.5}[/tex]

Cross-multiply.

x=6.5tan62°

x = 12.22 m

Height of the tree = 1.6 m + 12.22m

Height of the tree = 13.82 m

OR

Height of the tree = 1382 cm approximately.

Me.Hoffman has a doorstop in his classroom shaped like a triangular prism shown

Answers

- To determine the perimeter of the base, consider that the length is 5 in and the width is the same as the width of the top face of the prism, that is, 2 in. Then, the perimeters is:

P = 2l + 2w

w = 2 in

l = 5 in

P = 2(5 in) + 2(2 in)

P = 10 in + 4 in

P = 14 in

- The height of the doorstop is 1.2 in

- The area of the base is:

A = wl

A = (2 in)(5 in)

A = 10 in²

Which parabola corresponds to the quadratic function y = 2x2 + 4x - 16? D. A. B. C. 10:13 1618 10- 12 =10 10 28 -20

Answers

We can see that the y-intercept would be (0,-16) since this is the result of replacing x=0 in the function.

We can also find the x-intercepts solving the equation 0=2x^2+4x-16. Doing so, we have:

[tex]\begin{gathered} 0=2x^2+4x-16 \\ 0=x^2+2x-8\text{ (Dividing by 2 on both sides of the equation)} \\ 0=(x+4)(x-2)\text{ (Factoring)} \\ \text{ We can see that the solutions of the equation are x=-4 and x=2} \\ \text{Therefore the x-intercepts are (-4,0) and (2,0)} \end{gathered}[/tex]

The graph that satisfies the conditions we have found previously is the option A.

For each ordered pair, determine whether it is a solution to 4x - 5y = -13.Is it a solution?x$(x, y)YesNo(-7, -3)(3, -4)OO(-2, 1)oO(6, 7)0

Answers

The equation is 4x - 5y = -13.

Substitute -7 for x and -3 for y in the equation to check whether ordered pair is solution of the equation.

[tex]\begin{gathered} 4\cdot(-7)-5\cdot(-3)=-13 \\ -28+15=-13 \\ -13=-13 \end{gathered}[/tex]

The ordered pair satisfy the equation so point (-7,-3) is solution of equation.

Substitute 3 for x and -4 for y in the equation to check whether ordered pair is solution of the equation.

[tex]\begin{gathered} 4\cdot3-5\cdot(-4)=-13 \\ 12+20=-13 \\ 32\ne-13 \end{gathered}[/tex]

The ordered pair not satisfy the equation. So point (3,-4) is not a solution of the equation.

Substitute -2 for x and 1 for y in the equation to check whether ordered pair is solution of the equation.

[tex]\begin{gathered} 4\cdot(-2)-5\cdot1=-13 \\ -8-5=-13 \\ -13=-13 \end{gathered}[/tex]

The ordered pair satisfy the equation. So point (-2,1) is solution of equation.

Substitute 6 for x and 7 for y in the equation to check whether ordered pair is solution of the equation.

[tex]\begin{gathered} 4\cdot6-5\cdot7=-13 \\ 24-35=-13 \\ -11\ne-13 \end{gathered}[/tex]

The orderedpair not satisfy the equation. So point (6,7) is not a solution of the equation.

White the standard form of the equation of the line through the given point with the given slope.

Answers

The standard form equation of a line is expressed as

Ax + By = C

where

A, B and C are real numbers and A and B are not both zero. From the information given,

the line passes through(- 2, 5) and slope = - 4

We would find the y intercept of the line, c by substituting slope, m = - 4, x = - 2 and y = 5 into the slope intercept equation which is expressed as

y = mx + c

Thus, we have

5 = - 4 * - 2 + c

5 = 8 + c

c = 5 - 8 = - 3

Thus, the equation of the line in the slope intercept form is

y = - 4x - 3

We would convert it to standard form. Thus, we have

y + 4x = - 3

4x + y = - 3

Thus, the equation in standard form is

4x + y = - 3

what is 40+56 in GCF

Answers

The GCF stands for greatest common factor. To represent a sum by its GCF we need to use the distributive property and we need to first find the GCF of the numbers. Let's break each number by its factors:

[tex]\begin{gathered} 40=2\cdot2\cdot2\cdot5 \\ 56=2\cdot2\cdot2\cdot7 \end{gathered}[/tex]

We now multiply the numbers that appear on both.

[tex]\text{GCF}=2\cdot2\cdot2=8[/tex]

We now apply the distributive property:

[tex]8\cdot(5+7)[/tex]

what is 9932.8 rounded to the nearest integer

Answers

ANSWER

9933

EXPLANATION

We have the number 9932.8.

We want to round it to the nearest integer.

An integer is a number that can be written without decimal or fraction.

To do that, we follow the following steps:

1. Identify the number after the decimal

2. If the number is greater than or equal to 5, round up to 1 and add to the number before the decimal.

3. If the number is less than 5, round down to 0.

Since the number after the decimal is 8, we therefore have that:

[tex]9932.8\text{ }\approx\text{ 9933}[/tex]

What do you notice about the measures of the sides or the measures of angles that form triangles?

Answers

The angles sum up to give 180°

Only one of the angles can be an obtuse angle, we can;t have two bothuse angle in a triangle. BUT we can have two acute angles and one obtuse angle in a triangle.

We can also have a 90 degree and 2 acute angle in a triangle.

Examples

The angles sum up to give 180°

Only one of the angles can be an obtuse angle, we can;t have two bothuse angle in a triangle. BUT we can have two acute angles and one obtuse angle in a triangle.

We can also have a 90 degree and 2 acute angle in a triangle.

Examples

Other Questions
when harley-davidson advertises to reinforce the perception that the brand is masculine, rugged, and individualistic, it is building brand . multiple choice question. awareness value personality sensitivity What is a quadrilateral that has reflection symmetry, but not rotation symmetry? for general mills, it makes the most economical sense to produce large amounts of breakfast cereal each planning period. for the average consumer, it makes the most sense to buy a box of breakfast cereal when the consumer needs one. this difference is known as . I need help with #1 of this problem. It has writings on it because I just looked up the answer because Im confused but I want to know the answer and how to do it with work provided please 2. since she had her children 30 years ago, both of whom were 10lb babies, sally has hypertension and low hdl levels. are there any other diseases for which she also may be at high risk? The National Oceanic and Atmospheric Administration tracks the amount of oysters harvested from the Chesapeake Bay each year.Years since 1900 2 28 53 67 78 89Oysters (metric tons) 54.2 22.5 7.38 5.28 3.52 2.38Find the exponential regression equation that models this data.Ay=-58(-0.964)OB. y = -58(0.964)Oc.y=58(0.964)*OD.y=58(-0.964)*Reset SelectionPreviouNext Select all the situations in which a proportional relationship is described.Jackson saves $10 in the first month and $30 in the next 3 months.Mia saves $8 in the first 2 months and $4 in the next month.Piyoli spends $2 in the first 2 days of the week and $5 in the next 5 days.Robert spends $2 in the first 3 days of the week and $5 in the next 4 days. Hi Mr or Ms could you help me with this problem because I'm getting thrown off. it would be really awesome if you could help me out step by step here because I'm a bit stuck To make banana berry smoothies, Just Juice mixes water and juice in a ratio of 5 to 3. How much water should Just Juice mix with 23 gallons of juice to make banana berry smoothies? The sum of 4 times a number and 5 is the same as the difference of the number and 7 Explain whether or not Morgan had too much influence on the government.Support your answer with evidence from some type of text. the sales tax is 47 on the purchase of a dining room set for 940. find the sales tax rate. what is the maximum speed with which a 1200- kgkg car can round a turn of radius 95.0 mm on a flat road if the coefficient of static friction between tires and road is 0.60? 4Select the correct equations.Gracie, Mary, and Nancy each have a small collection of seashells.Gracie has 5 more thantimes the number of shells Mary has.Nancy has 1 more thantimes the number of shells Mary has.Gracie and Nancy have the same number of shells.Ifxis the number of shells Mary has, identify the equation that represents this situation and identify its solution. a potential cancer-causing gene coding for a protein with cell cycle control responsibilities is a(n) , and a gene coding for a protein that stimulates cell division is a . 10 N of force is applied to a spring and its length increases by 4 cm. If the spring is obeying Hooke's law, what will its extension be if 30 N of force is applied? Please help ASAP! Select all rational numbers. Find f such that the given conditions are satisfied. f(x)=x2-3x + 12, f(0) = 9 O f(x) = 1x2 - 4x + 12x +9 O O f(x) - x-x2 + 12x + 1 f(x) = 3x3-4x2 + 12x + 1 O f(x) = 3x - x? + 12x + 9 during keon's golf drive, the angular velocity of his club is zero at the top of the backswing and 33 rad/s at the bottom of the downswing just before ball impact. the downswing lasts 0.29 s, and the distance from the club head to the axis of rotation is 1.88 m at the bottom of the downswing. what is the average tangential acceleration of the club during the downswing (in m/s2)? QUICKLY PLEASE ITS DUE TOMORROW, IM OFFERING 100 POINTSThe deliberations of the Constitutional Convention of 1787 were held in strict secrecy. consequently, anxious citizens gathered outside Independence Hall when the proceedings ended in order to learn what had been produced behind closed doors. the answer was provided immediately. A Mrs. Powel of Philadelphia asked Benjamin Franklin, "Well, Doctor, what have we got, a republic or a monarchy?" With no hesitation whatsoever, Franklin responded, "A republic, if you can keep it." 21. Read the quote. What type of government was formed according to Benjamin Franklin?